Mathcenter Forum  

Go Back   Mathcenter Forum > คณิตศาสตร์โอลิมปิก และอุดมศึกษา > ข้อสอบโอลิมปิก
สมัครสมาชิก คู่มือการใช้ รายชื่อสมาชิก ปฏิทิน ข้อความวันนี้

ตั้งหัวข้อใหม่ Reply
 
เครื่องมือของหัวข้อ ค้นหาในหัวข้อนี้
  #256  
Old 12 เมษายน 2012, 17:44
BLACK-Dragon's Avatar
BLACK-Dragon BLACK-Dragon ไม่อยู่ในระบบ
กระบี่ประสานใจ
 
วันที่สมัครสมาชิก: 04 พฤศจิกายน 2010
ข้อความ: 719
BLACK-Dragon is on a distinguished road
Default

กำหนด c เป็นจำนวนตรรกยะ
จงแสดงว่า $x^3-3cx-3x+c$ มีรากเป็นจำนวนตรรกยะอย่างมากหน่ึงตัว
ตอบพร้อมอ้างอิงข้อความนี้
  #257  
Old 12 เมษายน 2012, 21:53
~ArT_Ty~'s Avatar
~ArT_Ty~ ~ArT_Ty~ ไม่อยู่ในระบบ
ลมปราณไร้สภาพ
 
วันที่สมัครสมาชิก: 03 กรกฎาคม 2010
ข้อความ: 1,081
~ArT_Ty~ is on a distinguished road
Default

อ้างอิง:
ข้อความเดิมเขียนโดยคุณ BLACK-Dragon View Post
กำหนด c เป็นจำนวนตรรกยะ
จงแสดงว่า $x^3-3cx-3x+c$ มีรากเป็นจำนวนตรรกยะอย่างมากหน่ึงตัว
กำหนด c เป็นจำนวนตรรกยะ
จงแสดงว่า $x^3-3cx^2-3x+c$ มีรากเป็นจำนวนตรรกยะอย่างมากหน่ึงตั
__________________
...สีชมพูจะไม่จางด้วยเหงื่อ แต่จะจางด้วยนํ้าลาย...
ตอบพร้อมอ้างอิงข้อความนี้
  #258  
Old 12 เมษายน 2012, 22:11
~ArT_Ty~'s Avatar
~ArT_Ty~ ~ArT_Ty~ ไม่อยู่ในระบบ
ลมปราณไร้สภาพ
 
วันที่สมัครสมาชิก: 03 กรกฎาคม 2010
ข้อความ: 1,081
~ArT_Ty~ is on a distinguished road
Default

อ้างอิง:
ข้อความเดิมเขียนโดยคุณ BLACK-Dragon View Post
Hint หน่อยครับ
มีวิธีสั้นๆแบบใช้ Inversion

แต่ผมอยากรู้วิธีแบบธรรมดาแบบที่ไม่ใช้ Advance Euclidean Geometry อ่ะครับ
__________________
...สีชมพูจะไม่จางด้วยเหงื่อ แต่จะจางด้วยนํ้าลาย...
ตอบพร้อมอ้างอิงข้อความนี้
  #259  
Old 13 เมษายน 2012, 15:31
จูกัดเหลียง's Avatar
จูกัดเหลียง จูกัดเหลียง ไม่อยู่ในระบบ
ลมปราณไร้สภาพ
 
วันที่สมัครสมาชิก: 21 กุมภาพันธ์ 2011
ข้อความ: 1,234
จูกัดเหลียง is on a distinguished road
Default

อ้างอิง:
กำหนด $c$ เป็นจำนวนตรรกยะ
จงแสดงว่า $x^3-3cx^2-3x+c$ มีรากเป็นจำนวนตรรกยะอย่างมากหนึ่งตัว
ไม่เเน่ใจนะครับ
สมมุติว่ามีรากตรรกยะ $3$ ราก ที่ชื่อ $x_1,x_2,x_3\in\mathbb{Q}$
ดังนั้น $x_1+x_2+x_3=3c,x_1x_2+x_2x_3+x_3x_1=-3,x_1x_2x_3=-c$
กรณีที่ $c>0$ ได้ว่า มีสองตัวใดๆที่เป็นบวกเเละอีกหนึ่งตัวเป็นลบ เพียงกรณีเดียวที่สอดคล้องสมการข้างต้น
โดยไม่เสียนัยให้ $x_1,x_2>0>x_3$ ดังนั้นโดย Cauchy จึงได้ว่า $$\frac{x_1x_2+x_2x_3+x_3x_1}{x_1x_2x_3}=\frac{3}{c}=\frac{1}{x_1}+\frac{1}{x_2}+\frac{1}{x_3}\ge \frac{4}{3c-x_3}+\frac{1}{x_3}=\frac{3x_3+3c}{x_3(3c-x_3)}$$ นั่นคือ $$\frac{x_3+c}{x_3(3c-x_3)}\le \frac{1}{c}\leftrightarrow (x_3-c)^2\le 0\therefore x_3=c$$
ซึ่งขัดเเย้ง เพราะ $0<c=x_3<0$
กรณีที่ $c<0$ สมมุติได้ว่า $x_1,x_2<0<x_3$ จาก $$\frac{3}{c}=\frac{1}{x_1}+\frac{1}{x_2}+\frac{1}{x_3}<\frac{1}{x_3}\rightarrow x_3<c/3$$ เเต่ $x_1+x_2+x_3=3c<x_3$ ดังนั้น $3c<x_3<c/3$ ขัดเเย้ง
เเละกรณีที่ มีรากตรรกยะ 2 รากก็เห็นได้ชัดว่าเป็นไปไม่ได้ ดังนั้น มี $x\in\mathbb{Q}$ ไม่เกิน(อย่างมาก) 1 ตัวที่สอดคล้อง
__________________
Vouloir c'est pouvoir
ตอบพร้อมอ้างอิงข้อความนี้
  #260  
Old 13 เมษายน 2012, 15:41
จูกัดเหลียง's Avatar
จูกัดเหลียง จูกัดเหลียง ไม่อยู่ในระบบ
ลมปราณไร้สภาพ
 
วันที่สมัครสมาชิก: 21 กุมภาพันธ์ 2011
ข้อความ: 1,234
จูกัดเหลียง is on a distinguished road
Default

มีมาเติมให้ครับ Let $a,b,c>0$
1.and $a+b+c=3$ Prove that $$a\sqrt{1+b^3}+b\sqrt{1+c^3}+c\sqrt{1+a^3}\le 5$$
2.Prove $$\frac{(a+b)(b+c)(c+a)}{abc}+\frac{ab+bc+ca}{a^2+b^2+c^2}\ge 9$$
3.and $a^2+b^2+c^2=3$ Prove $$\frac{1}{2-a}+\frac{1}{2-b}+\frac{1}{2-c}\ge 3$$
ปล.ผมอยากได้โจทย์หลากหลายหน่อยครับ เช่น NT,FE,Comb,Geo,...
__________________
Vouloir c'est pouvoir
ตอบพร้อมอ้างอิงข้อความนี้
  #261  
Old 13 เมษายน 2012, 20:44
AnDroMeDa's Avatar
AnDroMeDa AnDroMeDa ไม่อยู่ในระบบ
ลมปราณบริสุทธิ์
 
วันที่สมัครสมาชิก: 10 ตุลาคม 2011
ข้อความ: 114
AnDroMeDa is on a distinguished road
Default

อ้างอิง:
ข้อความเดิมเขียนโดยคุณ จูกัดเหลียง View Post
มีมาเติมให้ครับ Let $a,b,c>0$
1.and $a+b+c=3$ Prove that $$a\sqrt{1+b^3}+b\sqrt{1+c^3}+c\sqrt{1+a^3}\le 5$$
2.Prove $$\frac{(a+b)(b+c)(c+a)}{abc}+\frac{ab+bc+ca}{a^2+b^2+c^2}\ge 9$$
3.and $a^2+b^2+c^2=3$ Prove $$\frac{1}{2-a}+\frac{1}{2-b}+\frac{1}{2-c}\ge 3$$
ปล.ผมอยากได้โจทย์หลากหลายหน่อยครับ เช่น NT,FE,Comb,Geo,...


ผมว่าข้อเรขาของคุณ~ArT_Ty~นี่ไม่เบาเลยนะ ถ้าอัดตรีโกณนี่เน่ามากๆ แสดงวิธีที่ใช้ Inversion หน่อยสิครับ
ปล.เดี๋ยวผมมาตั้งโจทย์นะ
ตอบพร้อมอ้างอิงข้อความนี้
  #262  
Old 14 เมษายน 2012, 00:24
~ArT_Ty~'s Avatar
~ArT_Ty~ ~ArT_Ty~ ไม่อยู่ในระบบ
ลมปราณไร้สภาพ
 
วันที่สมัครสมาชิก: 03 กรกฎาคม 2010
ข้อความ: 1,081
~ArT_Ty~ is on a distinguished road
Default

ไว้คิดได้จะเอามาลงนะครับ แต่ที่ใช้ Inversion นี่คือผมเห็นเฉลยมาอ่ะครับ

แต่ผมอยากทำแบบใช้ความรู้แค่ สอวน. อ่ะครับ

มีหนังสือมาแจกครับ จากคุณอากิตติ(ขอบคุณมากครับ : ) )

http://www.mediafire.com/?e8687h41aus1v7h

Excursions in Geometry
__________________
...สีชมพูจะไม่จางด้วยเหงื่อ แต่จะจางด้วยนํ้าลาย...

14 เมษายน 2012 00:26 : ข้อความนี้ถูกแก้ไขแล้ว 2 ครั้ง, ครั้งล่าสุดโดยคุณ ~ArT_Ty~
ตอบพร้อมอ้างอิงข้อความนี้
  #263  
Old 14 เมษายน 2012, 04:35
AnDroMeDa's Avatar
AnDroMeDa AnDroMeDa ไม่อยู่ในระบบ
ลมปราณบริสุทธิ์
 
วันที่สมัครสมาชิก: 10 ตุลาคม 2011
ข้อความ: 114
AnDroMeDa is on a distinguished road
Default

1.จงพิสูจน์ว่ามีจำนวนเฉพาะที่อยู่ในรูป$\frac{x^2+x+1}{y} $เมื่อ$x,y\in \mathbb{N} $ อยู่เป็นจำนวนอนันต์

2.กำหนดให้$ABCD$เป็นสี่เหลี่ยมนูน เส้นแบ่งครึ่งมุม$ABC$ ตัดกับด้าน$CD$ ที่จุด$E$ ถ้า$\angle BCD=\angle CDA$จงพิสูจน์ว่า ถ้า$\angle AEB=90^{\circ} $แล้ว$AB=AD+BC$ และบทกลับจริงหรือไม่(นั่นคือถ้า$AB=AD+BC$ แล้ว$\angle AEB=90^{\circ} $)

3.ให้$f:\mathbb{R} \rightarrow\mathbb{R} $เป็นฟังก์ชันต่อเนื่องโดยสอดคล้องเงื่อนไข
(i) $f(2548)=2005$
(ii) $f(x)\bullet f(f(x))=1,\forall x\in \mathbb{R} $
จงหาค่าของ $f(\frac{2548}{2005} )$

4.ให้$ABC$เป็นสามเหลี่ยมโดยที่มุม $ABC$และมุม $BAC$เป็นมุมแหลม เส้นแบ่งครึ่งมุมภายในและภายนอกของมุม$BAC$ ตัวกับเส้นตรง $BC$ ที่จุด $D$ และ $E$ ตามลำดับ ให้ $O$ เป็นจุดศูนย์กลางวงกลมล้อมสามเหลี่ยม $ADE$ ถ้า $P$ เป็นจุดบนวงกลม $O$ จงแสดงว่า $\frac{BP}{PC} =\frac{OB}{OA} $

5.มี $f:\mathbb{N} \rightarrow \mathbb{N} $ หรือไม่ที่สอคล้องคล้องกับ $f(f(n-1))=f(n+1)-f(n),\forall n\in \mathbb{N},n\geqslant 2 $

6.ให้$a_1,a_2,...,a_{2n}$เป็นจำนวนเต็มที่แตกต่างกันที่ทำให้สมการ $(x-a_1)(x-a_2)...(x-a_{2n})=(-1)^n(n!)^2$ มีรากเป็นจำนวนเต็ม จงพิสูจน์ว่า $2n|(a_1+a_2+...+a_{2n})$

7.ให้จำนวนนับ8จำนวนที่มีค่าต่างกันและมีค่าน้อยกว่าหรือเท่ากับ15 จงแสดงว่ามีอย่างน้อย3คู่(ใน8จำนวนนี้)ที่ผลต่างที่เป็นบวกมีค่าเท่ากัน (โดยแต่ละคู่จะไม่ใช้ซ้ำ(disjoint set))
ตอบพร้อมอ้างอิงข้อความนี้
  #264  
Old 14 เมษายน 2012, 05:05
polsk133's Avatar
polsk133 polsk133 ไม่อยู่ในระบบ
กระบี่ไร้สภาพ
 
วันที่สมัครสมาชิก: 14 สิงหาคม 2011
ข้อความ: 1,873
polsk133 is on a distinguished road
Default

7.มาก่อนหน่อยนึงละกันครับ
ผลต่างที่เป็นบวกมี14แบบ(1,2,...,14)
เลือกมา8จำนวนผลต่างที่เป็นได้มี8เลืก2=28แบบ
โดยรังนก จะได้ว่ามีอย่างน้อย1รังที่มีนก2ตัว. แต่รัง14มีได้แค่คู่เดียว(1,15)

ดังนั้นเมื่อพิจารณาใน13รังที่เหลือจะได้ว่ามีอย่างน้อย1รังที่มีนกอย่างน้อย3ตัว

แต่ยังคิดไม่ออกเรื่องใช้ไม่ซ้ำกัน
__________________
เพจรวมโจทย์คอมบินาทอริกที่น่าสนใจ
https://www.facebook.com/combilegends
ตอบพร้อมอ้างอิงข้อความนี้
  #265  
Old 14 เมษายน 2012, 10:41
~ArT_Ty~'s Avatar
~ArT_Ty~ ~ArT_Ty~ ไม่อยู่ในระบบ
ลมปราณไร้สภาพ
 
วันที่สมัครสมาชิก: 03 กรกฎาคม 2010
ข้อความ: 1,081
~ArT_Ty~ is on a distinguished road
Default

ข้อ 2 ขาไป ลองวาดเส้นขนานซักเส้นดูนะครับ และพยายามหาส่วนของเส้นตรงที่ยาวเท่ากัน 3 เส้นให้ได้ครับ

ขากลับ ทำคล้ายๆเดิมครับ แต่ว่าพยายามหามุมฉากอีกมุมก่อน แล้วมุมที่เราต้องการจะตามมาครับ
(วิธีผมนะ )
__________________
...สีชมพูจะไม่จางด้วยเหงื่อ แต่จะจางด้วยนํ้าลาย...
ตอบพร้อมอ้างอิงข้อความนี้
  #266  
Old 14 เมษายน 2012, 12:46
จูกัดเหลียง's Avatar
จูกัดเหลียง จูกัดเหลียง ไม่อยู่ในระบบ
ลมปราณไร้สภาพ
 
วันที่สมัครสมาชิก: 21 กุมภาพันธ์ 2011
ข้อความ: 1,234
จูกัดเหลียง is on a distinguished road
Default

อ้างอิง:
ข้อความเดิมเขียนโดยคุณ AnDroMeDa View Post
2.กำหนดให้$ABCD$เป็นสี่เหลี่ยมนูน เส้นแบ่งครึ่งมุม$ABC$ ตัดกับด้าน$CD$ ที่จุด$E$ ถ้า$\angle BCD=\angle CDA$จงพิสูจน์ว่า ถ้า$\angle AEB=90^{\circ} $แล้ว$AB=AD+BC$ และบทกลับจริงหรือไม่(นั่นคือถ้า$AB=AD+BC$ แล้ว$\angle AEB=90^{\circ} $)
ผมได้เเค่ขาไปอ่ะครับ
Pytagoras $AB^2=AE^2+BE^2$ เเละกำหนด $A\hat BE=E\hat BC=\alpha,A\hat DC=E\hat CB=\beta$ เห็นได้ว่า $\sin\alpha =AE/AB,\cos\alpha =EB/AB$
โดยกฎของไซน์ ในสามเหลี่ยม $ADE$ $$\frac{AE}{\sin\beta}=\frac{AD}{\sin (-\pi/2+(\alpha+\beta))}=\frac{AD\cdot AB}{AE\sin\beta-EB\cos\beta}\leftrightarrow \cos\beta=\frac{\sin\beta(AE^2-AD\cdot AB)}{AE\cdot EB}$$
เเละในสามเหลี่ยม $EBC$ $$\frac{EB}{\sin\beta}=\frac{BC}{\sin(\pi-(\alpha+\beta))}=\frac{AB\cdot BC}{AE\cos\beta+EB\sin\beta}\leftrightarrow \cos\beta=\frac{\sin\beta(AB\cdot BC-EB^2)}{AE\cdot EB}$$
ดังนั้น $$\frac{\sin\beta(AE^2-AD\cdot AB)}{AE\cdot EB}=\frac{\sin\beta(AB\cdot BC-EB^2)}{AE\cdot EB}\leftrightarrow AB(BC+AD)=AE^2+EB^2=AB^2$$
นั่นคือ $AB=AD+BC$
__________________
Vouloir c'est pouvoir

14 เมษายน 2012 13:22 : ข้อความนี้ถูกแก้ไขแล้ว 1 ครั้ง, ครั้งล่าสุดโดยคุณ จูกัดเหลียง
ตอบพร้อมอ้างอิงข้อความนี้
  #267  
Old 14 เมษายน 2012, 17:00
No.Name No.Name ไม่อยู่ในระบบ
ลมปราณคุ้มครองร่าง
 
วันที่สมัครสมาชิก: 20 เมษายน 2011
ข้อความ: 323
No.Name is on a distinguished road
Default

ใบ้ว่า CEAX เป็น cyclic ครับ เม่ือ X เป็นจุดตัดของวงกลมกับ AB
__________________
no pain no gain
ตอบพร้อมอ้างอิงข้อความนี้
  #268  
Old 14 เมษายน 2012, 19:00
passer-by passer-by ไม่อยู่ในระบบ
ผู้พิทักษ์กฎทั่วไป
 
วันที่สมัครสมาชิก: 11 เมษายน 2005
ข้อความ: 1,442
passer-by is on a distinguished road
Default

ข้อ 4 ของคุณ Andromeda ถ้าใครรู้ harmonic จะง่ายมากเลยครับ
---------------------------------------------------------------------------

มาปล่อยโจทย์เพิ่มให้ครับ

Q1 (Russia) : มีจำนวนเต็มบวก a,b,c ที่ (a,b,c)=1 และ $ a+b |c ^2 \,\, , b+c|a ^2 \,\, , a+c |b ^2 $ หรือไม่

Q2 : กำหนดจำนวนเต็มบวก m ,n พิสูจน์ว่ามีจำนวนนับ a เป็นอนันต์ที่ $ m | \phi(a+i)\,\, ,\forall i = 0,1,..,n$

Q3: สามเหลี่ยมมุมแหลม ABC มี P เป็นจุดภายใน และ $ d_a,d_b,d_c$ เป็นระยะจาก P ไปยังด้าน BC,CA,AB ตามลำดับ พิสูจน์ $$ \sum d^2_a \geq \sum (PA \sin (\frac{A}{2}))^2 \geq \frac{1}{3} (\sum d_a)^2 $$

Q4 (AMM) : $a_1,a_2,...,a_n >0 $ และ $a_{n+1} = a_1 $ พิสูจน์ $$ \sum_{k=1}^n (\frac{a_k}{a_{k+1}})^{n-1} \geq (2\sum_{k=1}^n a_k \cdot \prod_{k=1}^n a_k^{-1/n}) -n $$

Q5 (Romania): หาจำนวนนับ n ทั้งหมดที่ $ n | p_1^2 +p_2^2+...+p_n^2 $ สำหรับจำนวนเฉพาะ $p_i$ ต่างกันที่ > 3

------------------------------------------------------------------------------------

โจทย์รอบนี้ ขอไม่เฉลย แต่จะให้แค่ Hint (ถ้ามีคนถามครับ)
__________________
เกษียณตัวเอง ปลายมิถุนายน 2557 แต่จะกลับมาเป็นครั้งคราว
ตอบพร้อมอ้างอิงข้อความนี้
  #269  
Old 14 เมษายน 2012, 19:18
No.Name No.Name ไม่อยู่ในระบบ
ลมปราณคุ้มครองร่าง
 
วันที่สมัครสมาชิก: 20 เมษายน 2011
ข้อความ: 323
No.Name is on a distinguished road
Default

เรขาบทกลับนะครับ

สร้าง BX=BC พิสูจน์ว่า ADEX เป็น cyclic ได้ก็จบ
__________________
no pain no gain
ตอบพร้อมอ้างอิงข้อความนี้
  #270  
Old 14 เมษายน 2012, 23:23
~ArT_Ty~'s Avatar
~ArT_Ty~ ~ArT_Ty~ ไม่อยู่ในระบบ
ลมปราณไร้สภาพ
 
วันที่สมัครสมาชิก: 03 กรกฎาคม 2010
ข้อความ: 1,081
~ArT_Ty~ is on a distinguished road
Default

ข้อ 4 ของคุณ Andromeda

ผมจะแสดงว่า locus ของจุด P ที่ทำให้ $\frac{BP}{PC}=\frac{BD}{DC}=\frac{BE}{EC}$ เมื่อ D,E เป็นจุดตัดของเส้นแบ่งครึ่งมุมภายในและภายนอก BAC กับ BC

สำหรับทุกจุดใดๆบน locus คือวงกลมล้อมรอบสามเหลี่ยม ADE

ก่อนอื่น จาก $\frac{BP}{PC}=\frac{BD}{DC}$ จะได้ว่า $PD,PE$ แบ่งครึ่งมุมภายในและภายนอกของมุม BPC ตามลำดับ

ดังนั้น มุม DPE มีขนาดเป็น 90 องศา เท่ากับมุม DAE จะได้ว่า D,P,A,E อยู่บนวงกลมเดียวกันคือวงกลมล้อมรอบสามเหลี่ยม DAE

และในทำนองเดียวกันกับจุดอื่นๆที่สอดคล้องกับเงื่อนไขดังกล่าวก็จะได้ว่า จุดทุกกจุดที่สอดคล้องจะอยู่บนวงกลมเดียวกันคือ

วงกลมล้อมรอบสามเหลี่ยม $ADE$ จึงทำให้ได้ว่า

ทุกจุดบนวงกลมล้อมรอบสามเหลี่ยม $ADE$ สอดคล้องกับเงื่อนไขดังกล่าว

หวังว่าน่าจะไปต่อได้นะครับ : ))
__________________
...สีชมพูจะไม่จางด้วยเหงื่อ แต่จะจางด้วยนํ้าลาย...

14 เมษายน 2012 23:24 : ข้อความนี้ถูกแก้ไขแล้ว 1 ครั้ง, ครั้งล่าสุดโดยคุณ ~ArT_Ty~
ตอบพร้อมอ้างอิงข้อความนี้
ตั้งหัวข้อใหม่ Reply



กฎการส่งข้อความ
คุณ ไม่สามารถ ตั้งหัวข้อใหม่ได้
คุณ ไม่สามารถ ตอบหัวข้อได้
คุณ ไม่สามารถ แนบไฟล์และเอกสารได้
คุณ ไม่สามารถ แก้ไขข้อความของคุณเองได้

vB code is On
Smilies are On
[IMG] code is On
HTML code is Off
ทางลัดสู่ห้อง


เวลาที่แสดงทั้งหมด เป็นเวลาที่ประเทศไทย (GMT +7) ขณะนี้เป็นเวลา 22:52


Powered by vBulletin® Copyright ©2000 - 2024, Jelsoft Enterprises Ltd.
Modified by Jetsada Karnpracha